Irreducible polynomial in $F[x]$ of degree $n$ is the minimal polynomial of exactly $n$ elements of $L$.












2














The problem goes like the following:




Let $F$ be a finite field, and $L/F$ an extension of degree $n$.



(a) Show that any irreducible polynomial in $F[x]$ of degree $n$ is the minimal polynomial of exactly $n$ elements of $L$.



(b) Suppose $|F|=p$. Determine, in terms of $p$, the number of irreducible polynomials in $F[x]$ of degree 3, and also the number of irreducible polynomials in $F[x]$ of degree 9, respectively.




My approach for (a) is: If $f(x)$ is an irreducible polynomial of degree $n$ in $F[x]$ and if $alphain L$ is a root of the polynomial, then we have $F(alpha)cong dfrac{F[x]}{(f(x))}$ and $[F(alpha):F]=n$. Since $[L:F]=n$, $L=F(alpha)$. I wanted to say that $L$ is Galois over $F$ so that it would contain all the roots of $f(x)$ and thus $f$ is irreducible and has exactly $n$ roots in $L$. But I don't know how to justify that $L/F$ is indeed a Galois extension. Also, it seems to me that if $|F|=p$, then $F=mathbb{F}_p$ and $L=mathbb{F}_{p^n}$ but I don't know if these would be helpful.



For part (b), using Mobius inversion formula, the number of irreducible polynomials in $F[x]$ of degree 3 is given by
$$frac{1}{3}sum_{d|3} mu(d)p^{3/d} = frac{1}{3}(p^3-p).$$
However, I don't think this would be the purpose of this problem. Now I've noted that $|L|=p^n$ as it can be viewed as a vector space over $F$ of dimension $n$. Using (a), since the irreducible polynomial in $F[x]$ of degree 3 is the minimal polynomial of 3 elements of $L$, there would be $dfrac{p^3}{3}$ such irreducible polynomials as $|L|=p^3$, which does not match with the result given by the Mobius inversion formula. Can someone help me out here? Many thanks!



Edit: I still need some help on proving part (a), though. If anyone has anything, please leave a comment or a solution. That would be really helpful!










share|cite|improve this question




















  • 1




    In your next-to-last sentence, you forgot to account for the $p$ elements of $L$ that are already in $F$. Their irreducibles are linear, and taking account of this, you again get a count of $(p^3-p)/3$
    – Lubin
    Jan 6 at 2:11










  • @Lubin Thanks for the comment. Yeah, I've thought about that. But when I was considering the number of irreducible polynomials in $F[x]$ of degree 9, I get $(p^9-p)/9$ by the same reasoning, while the Mobius inversion formula gives $(p^9-p^3)/9$. Am I missing something in this case?
    – Alex
    Jan 6 at 2:15








  • 2




    Well, once again, you have to exclude the elements in $Bbb F_{p^9}$ that are already in $Bbb F_{p^3}$. Remember that you want to count only the elements of the biggest field that aren’t in any smaller field.
    – Lubin
    Jan 6 at 2:22












  • @Lubin Ohhh, I see now. Thanks for pointing that out!
    – Alex
    Jan 6 at 2:55
















2














The problem goes like the following:




Let $F$ be a finite field, and $L/F$ an extension of degree $n$.



(a) Show that any irreducible polynomial in $F[x]$ of degree $n$ is the minimal polynomial of exactly $n$ elements of $L$.



(b) Suppose $|F|=p$. Determine, in terms of $p$, the number of irreducible polynomials in $F[x]$ of degree 3, and also the number of irreducible polynomials in $F[x]$ of degree 9, respectively.




My approach for (a) is: If $f(x)$ is an irreducible polynomial of degree $n$ in $F[x]$ and if $alphain L$ is a root of the polynomial, then we have $F(alpha)cong dfrac{F[x]}{(f(x))}$ and $[F(alpha):F]=n$. Since $[L:F]=n$, $L=F(alpha)$. I wanted to say that $L$ is Galois over $F$ so that it would contain all the roots of $f(x)$ and thus $f$ is irreducible and has exactly $n$ roots in $L$. But I don't know how to justify that $L/F$ is indeed a Galois extension. Also, it seems to me that if $|F|=p$, then $F=mathbb{F}_p$ and $L=mathbb{F}_{p^n}$ but I don't know if these would be helpful.



For part (b), using Mobius inversion formula, the number of irreducible polynomials in $F[x]$ of degree 3 is given by
$$frac{1}{3}sum_{d|3} mu(d)p^{3/d} = frac{1}{3}(p^3-p).$$
However, I don't think this would be the purpose of this problem. Now I've noted that $|L|=p^n$ as it can be viewed as a vector space over $F$ of dimension $n$. Using (a), since the irreducible polynomial in $F[x]$ of degree 3 is the minimal polynomial of 3 elements of $L$, there would be $dfrac{p^3}{3}$ such irreducible polynomials as $|L|=p^3$, which does not match with the result given by the Mobius inversion formula. Can someone help me out here? Many thanks!



Edit: I still need some help on proving part (a), though. If anyone has anything, please leave a comment or a solution. That would be really helpful!










share|cite|improve this question




















  • 1




    In your next-to-last sentence, you forgot to account for the $p$ elements of $L$ that are already in $F$. Their irreducibles are linear, and taking account of this, you again get a count of $(p^3-p)/3$
    – Lubin
    Jan 6 at 2:11










  • @Lubin Thanks for the comment. Yeah, I've thought about that. But when I was considering the number of irreducible polynomials in $F[x]$ of degree 9, I get $(p^9-p)/9$ by the same reasoning, while the Mobius inversion formula gives $(p^9-p^3)/9$. Am I missing something in this case?
    – Alex
    Jan 6 at 2:15








  • 2




    Well, once again, you have to exclude the elements in $Bbb F_{p^9}$ that are already in $Bbb F_{p^3}$. Remember that you want to count only the elements of the biggest field that aren’t in any smaller field.
    – Lubin
    Jan 6 at 2:22












  • @Lubin Ohhh, I see now. Thanks for pointing that out!
    – Alex
    Jan 6 at 2:55














2












2








2







The problem goes like the following:




Let $F$ be a finite field, and $L/F$ an extension of degree $n$.



(a) Show that any irreducible polynomial in $F[x]$ of degree $n$ is the minimal polynomial of exactly $n$ elements of $L$.



(b) Suppose $|F|=p$. Determine, in terms of $p$, the number of irreducible polynomials in $F[x]$ of degree 3, and also the number of irreducible polynomials in $F[x]$ of degree 9, respectively.




My approach for (a) is: If $f(x)$ is an irreducible polynomial of degree $n$ in $F[x]$ and if $alphain L$ is a root of the polynomial, then we have $F(alpha)cong dfrac{F[x]}{(f(x))}$ and $[F(alpha):F]=n$. Since $[L:F]=n$, $L=F(alpha)$. I wanted to say that $L$ is Galois over $F$ so that it would contain all the roots of $f(x)$ and thus $f$ is irreducible and has exactly $n$ roots in $L$. But I don't know how to justify that $L/F$ is indeed a Galois extension. Also, it seems to me that if $|F|=p$, then $F=mathbb{F}_p$ and $L=mathbb{F}_{p^n}$ but I don't know if these would be helpful.



For part (b), using Mobius inversion formula, the number of irreducible polynomials in $F[x]$ of degree 3 is given by
$$frac{1}{3}sum_{d|3} mu(d)p^{3/d} = frac{1}{3}(p^3-p).$$
However, I don't think this would be the purpose of this problem. Now I've noted that $|L|=p^n$ as it can be viewed as a vector space over $F$ of dimension $n$. Using (a), since the irreducible polynomial in $F[x]$ of degree 3 is the minimal polynomial of 3 elements of $L$, there would be $dfrac{p^3}{3}$ such irreducible polynomials as $|L|=p^3$, which does not match with the result given by the Mobius inversion formula. Can someone help me out here? Many thanks!



Edit: I still need some help on proving part (a), though. If anyone has anything, please leave a comment or a solution. That would be really helpful!










share|cite|improve this question















The problem goes like the following:




Let $F$ be a finite field, and $L/F$ an extension of degree $n$.



(a) Show that any irreducible polynomial in $F[x]$ of degree $n$ is the minimal polynomial of exactly $n$ elements of $L$.



(b) Suppose $|F|=p$. Determine, in terms of $p$, the number of irreducible polynomials in $F[x]$ of degree 3, and also the number of irreducible polynomials in $F[x]$ of degree 9, respectively.




My approach for (a) is: If $f(x)$ is an irreducible polynomial of degree $n$ in $F[x]$ and if $alphain L$ is a root of the polynomial, then we have $F(alpha)cong dfrac{F[x]}{(f(x))}$ and $[F(alpha):F]=n$. Since $[L:F]=n$, $L=F(alpha)$. I wanted to say that $L$ is Galois over $F$ so that it would contain all the roots of $f(x)$ and thus $f$ is irreducible and has exactly $n$ roots in $L$. But I don't know how to justify that $L/F$ is indeed a Galois extension. Also, it seems to me that if $|F|=p$, then $F=mathbb{F}_p$ and $L=mathbb{F}_{p^n}$ but I don't know if these would be helpful.



For part (b), using Mobius inversion formula, the number of irreducible polynomials in $F[x]$ of degree 3 is given by
$$frac{1}{3}sum_{d|3} mu(d)p^{3/d} = frac{1}{3}(p^3-p).$$
However, I don't think this would be the purpose of this problem. Now I've noted that $|L|=p^n$ as it can be viewed as a vector space over $F$ of dimension $n$. Using (a), since the irreducible polynomial in $F[x]$ of degree 3 is the minimal polynomial of 3 elements of $L$, there would be $dfrac{p^3}{3}$ such irreducible polynomials as $|L|=p^3$, which does not match with the result given by the Mobius inversion formula. Can someone help me out here? Many thanks!



Edit: I still need some help on proving part (a), though. If anyone has anything, please leave a comment or a solution. That would be really helpful!







abstract-algebra galois-theory finite-fields






share|cite|improve this question















share|cite|improve this question













share|cite|improve this question




share|cite|improve this question








edited 2 days ago







Alex

















asked Jan 5 at 23:23









AlexAlex

517




517








  • 1




    In your next-to-last sentence, you forgot to account for the $p$ elements of $L$ that are already in $F$. Their irreducibles are linear, and taking account of this, you again get a count of $(p^3-p)/3$
    – Lubin
    Jan 6 at 2:11










  • @Lubin Thanks for the comment. Yeah, I've thought about that. But when I was considering the number of irreducible polynomials in $F[x]$ of degree 9, I get $(p^9-p)/9$ by the same reasoning, while the Mobius inversion formula gives $(p^9-p^3)/9$. Am I missing something in this case?
    – Alex
    Jan 6 at 2:15








  • 2




    Well, once again, you have to exclude the elements in $Bbb F_{p^9}$ that are already in $Bbb F_{p^3}$. Remember that you want to count only the elements of the biggest field that aren’t in any smaller field.
    – Lubin
    Jan 6 at 2:22












  • @Lubin Ohhh, I see now. Thanks for pointing that out!
    – Alex
    Jan 6 at 2:55














  • 1




    In your next-to-last sentence, you forgot to account for the $p$ elements of $L$ that are already in $F$. Their irreducibles are linear, and taking account of this, you again get a count of $(p^3-p)/3$
    – Lubin
    Jan 6 at 2:11










  • @Lubin Thanks for the comment. Yeah, I've thought about that. But when I was considering the number of irreducible polynomials in $F[x]$ of degree 9, I get $(p^9-p)/9$ by the same reasoning, while the Mobius inversion formula gives $(p^9-p^3)/9$. Am I missing something in this case?
    – Alex
    Jan 6 at 2:15








  • 2




    Well, once again, you have to exclude the elements in $Bbb F_{p^9}$ that are already in $Bbb F_{p^3}$. Remember that you want to count only the elements of the biggest field that aren’t in any smaller field.
    – Lubin
    Jan 6 at 2:22












  • @Lubin Ohhh, I see now. Thanks for pointing that out!
    – Alex
    Jan 6 at 2:55








1




1




In your next-to-last sentence, you forgot to account for the $p$ elements of $L$ that are already in $F$. Their irreducibles are linear, and taking account of this, you again get a count of $(p^3-p)/3$
– Lubin
Jan 6 at 2:11




In your next-to-last sentence, you forgot to account for the $p$ elements of $L$ that are already in $F$. Their irreducibles are linear, and taking account of this, you again get a count of $(p^3-p)/3$
– Lubin
Jan 6 at 2:11












@Lubin Thanks for the comment. Yeah, I've thought about that. But when I was considering the number of irreducible polynomials in $F[x]$ of degree 9, I get $(p^9-p)/9$ by the same reasoning, while the Mobius inversion formula gives $(p^9-p^3)/9$. Am I missing something in this case?
– Alex
Jan 6 at 2:15






@Lubin Thanks for the comment. Yeah, I've thought about that. But when I was considering the number of irreducible polynomials in $F[x]$ of degree 9, I get $(p^9-p)/9$ by the same reasoning, while the Mobius inversion formula gives $(p^9-p^3)/9$. Am I missing something in this case?
– Alex
Jan 6 at 2:15






2




2




Well, once again, you have to exclude the elements in $Bbb F_{p^9}$ that are already in $Bbb F_{p^3}$. Remember that you want to count only the elements of the biggest field that aren’t in any smaller field.
– Lubin
Jan 6 at 2:22






Well, once again, you have to exclude the elements in $Bbb F_{p^9}$ that are already in $Bbb F_{p^3}$. Remember that you want to count only the elements of the biggest field that aren’t in any smaller field.
– Lubin
Jan 6 at 2:22














@Lubin Ohhh, I see now. Thanks for pointing that out!
– Alex
Jan 6 at 2:55




@Lubin Ohhh, I see now. Thanks for pointing that out!
– Alex
Jan 6 at 2:55










1 Answer
1






active

oldest

votes


















0














Well, since no solution so far has been posted, I'll give what I figured out.



First, Part (a):



Proof: Suppose $|F| = q$ (note that here $q$ is not necessarily a prime so in turn not necessarily a prime subfield of a finite field, it could be a power of prime, since any finite field is isomorphic to $mathbb{F}_{p^r}$ for some prime $p$ and integre $rge 1$), and suppose $L/F$ is an extension of degree $n$, i.e. $[L:F]=n$. Viewing $L$ as a vector space over $F$ of dimension $n$, we see that $|L| = q^n$. This says that $L cong mathbb{F}_{q^n}$. We claim that $L/F$ is a Galois extension. Note that $L^{times}$ (is cyclic being the mutiplicative group of $L$, which is finite) has order $q^n-1$. So for any $thetain L$,
$$theta^{q^n-1}=1implies theta^{q^n}=thetaimplies text{$theta$ is a root of the separable polynomial (by derivative test) $x^{q^n}-x$ over $F$.} $$
Therefore, $L$ is a subfield of the splitting field of $x^{q^n}-x$ hence is the splitting field as $|L|=q^n$. So such $L$ (an extension of $F$) exists and is Galois (also we have, by definition, $|$Gal$(L/F)|=[L:F]=n$).



Now suppose that $f(x)in F[x]$ is an irreducible polynomial of degree $n$ having $alphain Lsetminus F$ as a root. Then all roots of $f$ are contained in $L$ since the extension is Galois. These $n$ roots are distinct elements in $L$ since $f$ is irreducible over a perfect field (namely, the finite field $F$) thus separable. It then follows that $f$ is the irreducible polynomial for the $n$ roots in $L$ and so $f$ is the minimal polynomial of $n$ elements of $L$. (a) is proved.



Part (b) is solved (see the comments given by Lubin above).






share|cite|improve this answer





















    Your Answer





    StackExchange.ifUsing("editor", function () {
    return StackExchange.using("mathjaxEditing", function () {
    StackExchange.MarkdownEditor.creationCallbacks.add(function (editor, postfix) {
    StackExchange.mathjaxEditing.prepareWmdForMathJax(editor, postfix, [["$", "$"], ["\\(","\\)"]]);
    });
    });
    }, "mathjax-editing");

    StackExchange.ready(function() {
    var channelOptions = {
    tags: "".split(" "),
    id: "69"
    };
    initTagRenderer("".split(" "), "".split(" "), channelOptions);

    StackExchange.using("externalEditor", function() {
    // Have to fire editor after snippets, if snippets enabled
    if (StackExchange.settings.snippets.snippetsEnabled) {
    StackExchange.using("snippets", function() {
    createEditor();
    });
    }
    else {
    createEditor();
    }
    });

    function createEditor() {
    StackExchange.prepareEditor({
    heartbeatType: 'answer',
    autoActivateHeartbeat: false,
    convertImagesToLinks: true,
    noModals: true,
    showLowRepImageUploadWarning: true,
    reputationToPostImages: 10,
    bindNavPrevention: true,
    postfix: "",
    imageUploader: {
    brandingHtml: "Powered by u003ca class="icon-imgur-white" href="https://imgur.com/"u003eu003c/au003e",
    contentPolicyHtml: "User contributions licensed under u003ca href="https://creativecommons.org/licenses/by-sa/3.0/"u003ecc by-sa 3.0 with attribution requiredu003c/au003e u003ca href="https://stackoverflow.com/legal/content-policy"u003e(content policy)u003c/au003e",
    allowUrls: true
    },
    noCode: true, onDemand: true,
    discardSelector: ".discard-answer"
    ,immediatelyShowMarkdownHelp:true
    });


    }
    });














    draft saved

    draft discarded


















    StackExchange.ready(
    function () {
    StackExchange.openid.initPostLogin('.new-post-login', 'https%3a%2f%2fmath.stackexchange.com%2fquestions%2f3063313%2firreducible-polynomial-in-fx-of-degree-n-is-the-minimal-polynomial-of-exac%23new-answer', 'question_page');
    }
    );

    Post as a guest















    Required, but never shown

























    1 Answer
    1






    active

    oldest

    votes








    1 Answer
    1






    active

    oldest

    votes









    active

    oldest

    votes






    active

    oldest

    votes









    0














    Well, since no solution so far has been posted, I'll give what I figured out.



    First, Part (a):



    Proof: Suppose $|F| = q$ (note that here $q$ is not necessarily a prime so in turn not necessarily a prime subfield of a finite field, it could be a power of prime, since any finite field is isomorphic to $mathbb{F}_{p^r}$ for some prime $p$ and integre $rge 1$), and suppose $L/F$ is an extension of degree $n$, i.e. $[L:F]=n$. Viewing $L$ as a vector space over $F$ of dimension $n$, we see that $|L| = q^n$. This says that $L cong mathbb{F}_{q^n}$. We claim that $L/F$ is a Galois extension. Note that $L^{times}$ (is cyclic being the mutiplicative group of $L$, which is finite) has order $q^n-1$. So for any $thetain L$,
    $$theta^{q^n-1}=1implies theta^{q^n}=thetaimplies text{$theta$ is a root of the separable polynomial (by derivative test) $x^{q^n}-x$ over $F$.} $$
    Therefore, $L$ is a subfield of the splitting field of $x^{q^n}-x$ hence is the splitting field as $|L|=q^n$. So such $L$ (an extension of $F$) exists and is Galois (also we have, by definition, $|$Gal$(L/F)|=[L:F]=n$).



    Now suppose that $f(x)in F[x]$ is an irreducible polynomial of degree $n$ having $alphain Lsetminus F$ as a root. Then all roots of $f$ are contained in $L$ since the extension is Galois. These $n$ roots are distinct elements in $L$ since $f$ is irreducible over a perfect field (namely, the finite field $F$) thus separable. It then follows that $f$ is the irreducible polynomial for the $n$ roots in $L$ and so $f$ is the minimal polynomial of $n$ elements of $L$. (a) is proved.



    Part (b) is solved (see the comments given by Lubin above).






    share|cite|improve this answer


























      0














      Well, since no solution so far has been posted, I'll give what I figured out.



      First, Part (a):



      Proof: Suppose $|F| = q$ (note that here $q$ is not necessarily a prime so in turn not necessarily a prime subfield of a finite field, it could be a power of prime, since any finite field is isomorphic to $mathbb{F}_{p^r}$ for some prime $p$ and integre $rge 1$), and suppose $L/F$ is an extension of degree $n$, i.e. $[L:F]=n$. Viewing $L$ as a vector space over $F$ of dimension $n$, we see that $|L| = q^n$. This says that $L cong mathbb{F}_{q^n}$. We claim that $L/F$ is a Galois extension. Note that $L^{times}$ (is cyclic being the mutiplicative group of $L$, which is finite) has order $q^n-1$. So for any $thetain L$,
      $$theta^{q^n-1}=1implies theta^{q^n}=thetaimplies text{$theta$ is a root of the separable polynomial (by derivative test) $x^{q^n}-x$ over $F$.} $$
      Therefore, $L$ is a subfield of the splitting field of $x^{q^n}-x$ hence is the splitting field as $|L|=q^n$. So such $L$ (an extension of $F$) exists and is Galois (also we have, by definition, $|$Gal$(L/F)|=[L:F]=n$).



      Now suppose that $f(x)in F[x]$ is an irreducible polynomial of degree $n$ having $alphain Lsetminus F$ as a root. Then all roots of $f$ are contained in $L$ since the extension is Galois. These $n$ roots are distinct elements in $L$ since $f$ is irreducible over a perfect field (namely, the finite field $F$) thus separable. It then follows that $f$ is the irreducible polynomial for the $n$ roots in $L$ and so $f$ is the minimal polynomial of $n$ elements of $L$. (a) is proved.



      Part (b) is solved (see the comments given by Lubin above).






      share|cite|improve this answer
























        0












        0








        0






        Well, since no solution so far has been posted, I'll give what I figured out.



        First, Part (a):



        Proof: Suppose $|F| = q$ (note that here $q$ is not necessarily a prime so in turn not necessarily a prime subfield of a finite field, it could be a power of prime, since any finite field is isomorphic to $mathbb{F}_{p^r}$ for some prime $p$ and integre $rge 1$), and suppose $L/F$ is an extension of degree $n$, i.e. $[L:F]=n$. Viewing $L$ as a vector space over $F$ of dimension $n$, we see that $|L| = q^n$. This says that $L cong mathbb{F}_{q^n}$. We claim that $L/F$ is a Galois extension. Note that $L^{times}$ (is cyclic being the mutiplicative group of $L$, which is finite) has order $q^n-1$. So for any $thetain L$,
        $$theta^{q^n-1}=1implies theta^{q^n}=thetaimplies text{$theta$ is a root of the separable polynomial (by derivative test) $x^{q^n}-x$ over $F$.} $$
        Therefore, $L$ is a subfield of the splitting field of $x^{q^n}-x$ hence is the splitting field as $|L|=q^n$. So such $L$ (an extension of $F$) exists and is Galois (also we have, by definition, $|$Gal$(L/F)|=[L:F]=n$).



        Now suppose that $f(x)in F[x]$ is an irreducible polynomial of degree $n$ having $alphain Lsetminus F$ as a root. Then all roots of $f$ are contained in $L$ since the extension is Galois. These $n$ roots are distinct elements in $L$ since $f$ is irreducible over a perfect field (namely, the finite field $F$) thus separable. It then follows that $f$ is the irreducible polynomial for the $n$ roots in $L$ and so $f$ is the minimal polynomial of $n$ elements of $L$. (a) is proved.



        Part (b) is solved (see the comments given by Lubin above).






        share|cite|improve this answer












        Well, since no solution so far has been posted, I'll give what I figured out.



        First, Part (a):



        Proof: Suppose $|F| = q$ (note that here $q$ is not necessarily a prime so in turn not necessarily a prime subfield of a finite field, it could be a power of prime, since any finite field is isomorphic to $mathbb{F}_{p^r}$ for some prime $p$ and integre $rge 1$), and suppose $L/F$ is an extension of degree $n$, i.e. $[L:F]=n$. Viewing $L$ as a vector space over $F$ of dimension $n$, we see that $|L| = q^n$. This says that $L cong mathbb{F}_{q^n}$. We claim that $L/F$ is a Galois extension. Note that $L^{times}$ (is cyclic being the mutiplicative group of $L$, which is finite) has order $q^n-1$. So for any $thetain L$,
        $$theta^{q^n-1}=1implies theta^{q^n}=thetaimplies text{$theta$ is a root of the separable polynomial (by derivative test) $x^{q^n}-x$ over $F$.} $$
        Therefore, $L$ is a subfield of the splitting field of $x^{q^n}-x$ hence is the splitting field as $|L|=q^n$. So such $L$ (an extension of $F$) exists and is Galois (also we have, by definition, $|$Gal$(L/F)|=[L:F]=n$).



        Now suppose that $f(x)in F[x]$ is an irreducible polynomial of degree $n$ having $alphain Lsetminus F$ as a root. Then all roots of $f$ are contained in $L$ since the extension is Galois. These $n$ roots are distinct elements in $L$ since $f$ is irreducible over a perfect field (namely, the finite field $F$) thus separable. It then follows that $f$ is the irreducible polynomial for the $n$ roots in $L$ and so $f$ is the minimal polynomial of $n$ elements of $L$. (a) is proved.



        Part (b) is solved (see the comments given by Lubin above).







        share|cite|improve this answer












        share|cite|improve this answer



        share|cite|improve this answer










        answered 2 days ago









        AlexAlex

        517




        517






























            draft saved

            draft discarded




















































            Thanks for contributing an answer to Mathematics Stack Exchange!


            • Please be sure to answer the question. Provide details and share your research!

            But avoid



            • Asking for help, clarification, or responding to other answers.

            • Making statements based on opinion; back them up with references or personal experience.


            Use MathJax to format equations. MathJax reference.


            To learn more, see our tips on writing great answers.





            Some of your past answers have not been well-received, and you're in danger of being blocked from answering.


            Please pay close attention to the following guidance:


            • Please be sure to answer the question. Provide details and share your research!

            But avoid



            • Asking for help, clarification, or responding to other answers.

            • Making statements based on opinion; back them up with references or personal experience.


            To learn more, see our tips on writing great answers.




            draft saved


            draft discarded














            StackExchange.ready(
            function () {
            StackExchange.openid.initPostLogin('.new-post-login', 'https%3a%2f%2fmath.stackexchange.com%2fquestions%2f3063313%2firreducible-polynomial-in-fx-of-degree-n-is-the-minimal-polynomial-of-exac%23new-answer', 'question_page');
            }
            );

            Post as a guest















            Required, but never shown





















































            Required, but never shown














            Required, but never shown












            Required, but never shown







            Required, but never shown

































            Required, but never shown














            Required, but never shown












            Required, but never shown







            Required, but never shown







            Popular posts from this blog

            Mario Kart Wii

            What does “Dominus providebit” mean?

            Antonio Litta Visconti Arese